find the amount on 800 for one year at 10%​

Answers

Answer 1

Answer:

80

Step-by-step explanation:

You mean the "interest on"

I=800×.1×1=80

Answer 2
Percentage is 10% 800 =80
80/800= 10%
10% of 800=800

Related Questions

a+in=√1+i÷1-i,prove that a^2+b^2=1

Answers

Answer with Step-by-step explanation:

We are given that

[tex]a+ib=\sqrt{\frac{1+i}{1-i}}[/tex]

We have to prove that

[tex]a^2+b^2=1[/tex]

[tex]a+ib=\sqrt{\frac{(1+i)(1+i)}{(1-i)(1+i)}}[/tex]

Using rationalization property

[tex]a+ib=\sqrt{\frac{(1+i)^2}{(1^2-i^2)}}[/tex]

Using the property

[tex](a+b)(a-b)=a^2-b^2[/tex]

[tex]a+ib=\sqrt{\frac{(1+i)^2}{(1-(-1))}}[/tex]

Using

[tex]i^2=-1[/tex]

[tex]a+ib=\frac{1+i}{\sqrt{2}}[/tex]

[tex]a+ib=\frac{1}{\sqrt{2}}+i\frac{1}{\sqrt{2}}[/tex]

By comparing we get

[tex]a=\frac{1}{\sqrt{2}}, b=\frac{1}{\sqrt{2}}[/tex]

[tex]a^2+b^2=(\frac{1}{\sqrt{2}})^2+(\frac{1}{\sqrt{2}})^2[/tex]

[tex]a^2+b^2=\frac{1}{2}+\frac{1}{2}[/tex]

[tex]a^2+b^2=\frac{1+1}{2}[/tex]

[tex]a^2+b^2=\frac{2}{2}[/tex]

[tex]a^2+b^2=1[/tex]

Hence, proved.

Is the random variable described discrete or continuous? The amount of rain during the next thunderstorm.

Answers

Answer:

continuous

rain does not fall in specific units like 1 inch , 2 inches etc... but 1.23456 etc..

Step-by-step explanation:

Barry and Robin walk to Dunkin' Donuts each Saturday to meet for coffee and donuts. Barry walks the 2 miles from his house in 30 minutes and Robin walks the 3 miles from his house in 36 minutes. Find the unit rate in minutes per mile for Barry. Find the unit rate in minutes per mile for Robin. Who walks faster, Barry or Robin

Answers

Answer : barry

12<15

Barry's unit rate is 15 minutes per mile and Robin's unit rate is 12 minutes per mile

The unit rates of Barry and Robin

We have:

Barry

Distance = 2 miles

Time = 30 minutes

Unit rate = Time/Distance

Unit rate = 30 minutes/2 miles

Unit rate = 15 minutes per mile

Robin

Distance = 3 miles

Time = 36 minutes

Unit rate = Time/Distance

Unit rate = 36 minutes/3 miles

Unit rate = 12 minutes per mile

Hence, Barry's unit rate is 15 minutes per mile and Robin's unit rate is 12 minutes per mile

Who walk faster?

The unit rates mean that:

Barry covers 1 mile in 15 minutesRobin covers 1 mile in 12 minutes

Hence, Robin walks faster

Read more about unit rates at:https://brainly.com/question/19493296

#SPJ6

what is the equation of the directx for the following parabola -8(x-5)=(y+1)^2

Answers

Answer:

x=7

Step-by-step explanation:

The directrix of a parabola is the vertical line found by subtracting

p from the x-coordinate h

of the vertex if the parabola opens left or right.

x=h-p

Substitute the known values of

p and h

into the formula and simplify.

x=7

suppose △abc≅△xyz. what is the corresponding congruent part for each segment or angle?​

Answers

Answer:

Step-by-step explanation:

Skylar's grades on four math tests are 85, 78, 77, and 69. What does Skylar need to score on the next test in order to have a mean score of 80?

Answers

Answer:

91

Step-by-step explanation:

The mean the the average of 5 numbers. If the next score is x, then the mean is:

(85 + 78 + 77 + 69 + x)/5 = 80

Solve it for x:

309 + x = 80*5x = 400 - 309x = 91

It is given that,

The mean is the average of 5 numbers.

Then if the,

Next score is x the mean will be.

We can solve now,

→ (85 +78 +77 + 69 + x)/5 = 80

→ (309 + x)/5 = 80

→ 309 + x = 80 × 5

→ 309 + x = 400

→ x = 400 - 309

→ x = 91

Hence, the next score is 91.

Can someone please help?
Find the value of x for the right triangle.

Answers

Answer:

5

Step-by-step explanation:

cos60° = x / 10

10cos60° = x

10cos60° = 5

What are the coordinates of vertex F" of ΔF"G"H"?

Answers

-b /2a into a equation for x and solve for y

Kaitlin is riding her bike. The number of revolutions (turns) her wheels make varies directly with the distance she travels. See the graph below.



How far does Kaitlin travel per revolution?

What is the slope of the graph?

Answers

Answer:

m = Δy/Δx = 4/32 = 1/8

8 ft/rev.

Step-by-step explanation:

Answer:

[tex]\frac{1}{8}[/tex] for both

Step-by-step explanation:

1. Distance per revolution

We see that the distance travelled at 2 revolutions is 16

This means that she travels [tex]\frac{1}{8}[/tex] of a mile per revolution

2. Slope

The slope formula is rise/run, or ([tex]y[/tex]₂[tex]-y[/tex]₁)/([tex]x[/tex]₂[tex]-x[/tex]₁)

We can take two points, (16,2) and (32,4)

[tex]\frac{4-2}{32-16}=\frac{2}{16}=\frac{1}{8}[/tex]

The slope is [tex]\frac{1}{8}[/tex]

Hope this helps! Please mark brainliest :)

Mr. Alvarado bought a total of 20 pounds of grass seed at the nursery for $168. He paid $9 per pound for Kentucky blue grass and $6 per pound for Tall Fescue. Which system of equations can be used to find the amount x (in pounds) of Kentucky blue grass and the amount y (in pounds) of Tall Fescue Mr. Alvarado purchased?

Answers

Answer:

K+T=20

$9K + $6T = $168

K is the Kentucky blue grass in pounds

T is the Tall fescue in pounds

Step-by-step explanation:

You can start with the first equation. We don't know the exact amounts of each but we know that there was a total of 20 pounds, and there were 2 types of grass seeds, so we can get that the amount of pounds of Kentucky blue grass(K) and the pounds of Tal Fescue(T) has a sum of 20.

K + T = 20

For the second equation we know that there is a sum of $168 so we'll start with that. Then, we know he paid $9 per pound of K so $9* the value of K is the amount paid for Kentucky blue grass total. This can be represented as 9K. We do the same for T, 6T. Since the sum of the cost of $9T and $6K must be $168 we can write this as:

$9K + $6T = $168

Use the definition of a Taylor series to find the first four nonzero terms of the series for f(x) centered at the given value of a. (Enter your answers as a comma-separated list.)
f(x) = 7/(1+x), a = 2
Find the Maclaurin series for f(x) using the definition of a Maclaurin series. [Assume that f has a power series expansion. Do not show that Rn(x) → 0.]
f(x) = e−5x
f(x)=
[infinity]
n = 0
=
Find the associated radius of convergence R.
R =

Answers

Answer:

A) [ 7/3,  (-7/9)(x/2),  7/27(x-2)^2,  (-7/81)(x-2)^3 ]

B) attached below

Step-by-step explanation:

A)  Using the definition of a Taylor series

The first four nonzero terms of the series for f(x) = 7/ (1 +x), a = 2

= [ 7/3,  (-7/9)(x/2),  7/27(x-2)^2,  (-7/81)(x-2)^3 ]

attached below is the detailed solution

B) Finding Maclaurin series for f(x)

f(x) = e^-5x

attached below

Associated radius of convergence = ∞  ( infinity )

A multiple regression model is ^Y = 8.114 + 2.005X1 + 0.774X2. Which of the following values is the estimate for the intercept parameter?
a. 0.774.
b. 8.114.
c. 1.000.
d. 2.005.

Answers

Answer:

B. 8.114

Step-by-step explanation:

The intercept parameter is the zero-grade component of the multilinear equation, that is, the component independent from [tex]x_{1}[/tex] and [tex]x_{2}[/tex]. Hence, the intercept parameter of the multilinear regression is 8.114. (Correct answer: B)

If (3x − 2)(3x + 2) = ax2 − b, what is the value of a?

Answers

(3x-2)(3x+ 2)

Multiply each term in one set of parentheses by the other terms:

3x x 3x =9x^2

3x x 2 = 6x

-2 x 3x = -6x

-2 x 2 = -4

Combine to get:

9x^2 + 6x - 6x -4

Combine like terms:

9x^2 - 4

The value of a would be 9.

[tex]\sf{\bold{\blue{\underline{\underline{Given}}}}}[/tex]

(3x − 2)(3x + 2) = ax2 − b⠀⠀⠀⠀

[tex]\sf{\bold{\red{\underline{\underline{To\:Find}}}}}[/tex]

⠀what is the value of a?⠀⠀⠀

[tex]\sf{\bold{\purple{\underline{\underline{Solution}}}}}[/tex]

At first we have to solve the value of (3x-2)(3x+2)

[tex]\sf{(3x-2)(3x+2) }[/tex]

[tex]\sf{3x(3x+2)-2(3x+2) }[/tex]

[tex]\sf{9x^{2}+6x-6x-4 }[/tex]

[tex]\sf{9x^{2}-4 }[/tex]

According to the question,

[tex]\sf{ (3x − 2)(3x + 2) = ax^{2} − b }[/tex]

[tex]\sf{9x^{2}-4=ax^{2}-b }[/tex]

[tex]\sf{9x^{2}=ax^{2}~and~-4=-b }[/tex]

[tex]\sf{a=9~and~b=4 }[/tex]

⠀⠀⠀

[tex]\sf{\bold{\green{\underline{\underline{Answer}}}}}[/tex]

Hence,

The value of a is 9

A radioactive material is known to decay at a yearly rate proportional to the amount at each moment. There were 1000 grams of the material 10 years ago. There are 980 grams right now. What will be the amount of the material right after 20 years

Answers

Answer:

x = 960.4

Step-by-step explanation:

980 = 1000[tex]e^{kt}[/tex]

.98 = [tex]e^{10 k}[/tex]

ln(.98) = 10k ln(e)

k = ln(.98)/10

k=-0.00202

~~~~~~~~~~~~~~

x = 1000[tex]e^{20 *-.00202}[/tex]

x = 960.4

The amount of the material right after 20 years will be x = 960.4.

What is an exponential expression?

Powers can simply be expressed in concise form using exponential expressions. The exponent shows how many times the base has been multiplied. Since 2 is the "base" and 5 is the "exponent," it can be represented as 2x2x2x2=25 for the number 32. This phrase should be understood as "two to the fifth power."

Given that radioactive material is known to decay at a yearly rate proportional to the amount at each moment. There were 1000 grams of the material 10 years ago. There are 980 grams right now.

The amount of the material will be calculated as,

980 = 1000

[tex]0.98 = e^{10k}[/tex]

ln(.98) = 10k ln(e)

k = ln(.98)/10

k=-0.00202

The value after 20 years will be,

[tex]x = 1000e^{20\times 0.00202}[/tex]

x = 960.4

Therefore, the amount of the material right after 20 years will be x = 960.4.

To know more about an exponential expression follow

https://brainly.com/question/2456547

#SPJ5

Which simplified fraction is equal to 0.53? Need answers now plz

Answers

Answer:

8/15

Step-by-step explanation:

Answer:

8/15

Step-by-step explanation:

when you divide 8/15 its 0.53

The number 55 is attached to a two-digit number to its left and the formed 4-digit number is divisible by 24. What could be the 2-digit number? List all options

Answers

the answer will be 44 I think I hoped I helped if not sorry.

8/9 - 1/3
Very easy question for 10 pts

Answers

Answer:

answer is 5/9

Step-by-step explanation:

Answer:

The answer is 5/9 or 0.555 (the 5 is repeated)

someone help and tell me what the correct answer is? got it wrong and want to learn​

Answers

The answer would be 48. Since UW is an angle bisector that would mean the angle was split in half evenly. Since the angles are now even, VUW = WUT. You can put the two equations equal to each other (4x + 6) = (6x - 10) after solving you would find that x = 8. Take that 8 and substitute it in the equation ( 6(8) - 10 ) and solve it.

In one U.S city, the taxi cost is $3 plus $0.80 per mile. If you are traveling from the airport, there is an additional charge of $5.50 for tolls. How far can you travel from the airport by taxi for $56.50?

Answers

Answer:

60 miles

Step-by-step explanation:

Create an equation where y is the total cost and x is the number of miles traveled.

0.8x will represent the cost from the miles traveled. 8.5 will be added to this to represent the taxi cost and additional charge from tolls:

y = 0.8x + 8.5

Plug in 56.50 as y and solve for x, the number of miles:

y = 0.8x + 8.5

56.5 = 0.8x + 8.5

48 = 0.8x

60 = x

So, you can travel 60 miles

The population of watesville decreases at a rate of 1.6% each year if the population was 62,500 in 2015 what will it be in 2021

Answers

Answer:

Step-by-step explanation:

We need to first find the model for this particular situation, knowing that this is an exponential decay problem. The main equation for exponential growth/decay (as far as population goes for our problem) is

[tex]y=a(b)^x[/tex] where a is the initial population, b is the rate of decrease in the population which can also be written as (1 - r), y is the population after a certain amount of time, x, goes by. We will let year 2015 = 0 so year 2021 can = 6. This keeps our numbers lower and doesn't change the answer!

Our initial population in the year x = 0 is 62500. Our rate of decay is

(1 - .016) so our b value is .984

Filling in to find our model:

[tex]y=62500(.984)^x[/tex]

Now we can use that model and sub in a 6 for x to find the population in the year 2021:

[tex]y=62500(.984)^6[/tex] and

y = 62500(.9077590568) so

y = 56734.9 or, rounded to the nearest person, 56735

Evaluate the expression

Answers

The exclamation mark at the end of any number indicates a factorial. You may be able to find this button in your calculator, though where it is varies.

A factorial is the product of the integer and all of the integers below it (greater than 0).

So, 9 factorial = 9 x 8 x 7 x 6 x 5 x 4 x 3 x 2 x 1

6 factorial = 6 x 5 x 4 x 3 x 2 x 1

We can immediately eliminate the numbers that are the same on the top and bottom, since this is a fraction. That leaves us with the following multiplication problem to solve.

9 x 8 x 7 = 504

Hope this helps!

PLEASE HELP!!! WILL GIVE BRAINLIEST!!!!!

Answers

Step-by-step explanation:

[tex]g(x) = 3^{\frac{x}{2}}[/tex]

For [tex]x = -2[/tex], we get

[tex]g(-2) = 3^{\frac{-2}{2}} = 3^{-1} = \frac{1}{3}[/tex]

GM projected that 3% of their cars produced this year will be defective. If GM produced 1,698 cars that were defective, how many cars did GM produce this year

Answers

Answer:

56600 cars

Step-by-step explanation:

Below is the calculation of number of cars produced.

The percentage of cars that is defected = 3%

Number of cars that are defective = 1698 cars

The number of cars produced in a year = 1698 / 3%  

The number of cars produced in a year = 56600 cars

2•^2=?

A) -4

B) 1/4

C) 4

Answers

B) 1/4
Explanation - I’m smart

Answer:

1/4.

Step-by-step explanation:

2^-2 = 1/2^2

= 1/4.

Help me pls, BRAINEST AWARD

Answers

Answer:

x = 3.7

Step-by-step explanation:

By applying sine ratio for the given angle B,

sin(39°) = [tex]\frac{\text{Opposite side}}{\text{Hypotenuse}}[/tex]

sin(39°) = [tex]\frac{AD}{AB}[/tex]

0.6293 = [tex]\frac{AD}{7}[/tex]

AD = 4.41

By applying tangent ratio for the given angle C,

tan(50°) = [tex]\frac{\text{Opposite side}}{\text{Adjacent side}}[/tex]

1.19 = [tex]\frac{AD}{x}[/tex]

1.19 = [tex]\frac{4.41}{x}[/tex]

x = 3.7

A boy leaves station X on a bearing of 035' to station Y. which is 21km away. He then travels to another station Z on a bearing of 125 degrees . If Z is directly East of X, what is the distance from X to his present position?​

Answers

9514 1404 393

Answer:

  36.6 km

Step-by-step explanation:

We assume the initial bearing of the boy is 35°. Then he will make a 90° turn to a heading of 125°. A diagram shows the distance of interest is the hypotenuse of a right triangle in which 35° is the angle opposite the side of length 21 km.

The relevant trig relation is ...

  Sin = Opposite/Hypotenuse

  sin(35°) = (21 km)/XZ

  XZ = (21 km)/sin(35°) ≈ 36.61 km

The distance from X to Z is about 36.61 km.

_____

The attached diagram has the angles measured in the usual way for a Cartesian plane: CCW from the +x axis. This will correspond to bearing measures if we relabel the axes so that +x is North, and +y is East.

Can someone help me out here? Not sure how to solve this problem or where to start either?

Answers

The paper is flying away

Answer:

19.3 miles per gallon

Step-by-step explanation:

First, subtract 54,042.8-53,737.7. The answer is 305.1

Then, find the unit rate. 305.1/15.8

You get 19.31012658. The prompt says to round to the nearest tenth, so round, and you get 19.3.

That's your answer!

The number of measles cases increased 26.3% to 321 cases this year. What was the number of cases prior to the increase? Express your answer rounded correctly to the nearest whole number.

Answers

Answer:

The right answer is "[tex]x\simeq 254[/tex]".

Step-by-step explanation:

Let the number of earlier case will be "x".

Now,

⇒ [tex]x+x\times \frac{26.3}{100}=321[/tex]

or,

⇒ [tex]x+x\times 0.263=321[/tex]

By taking "x" common, we get

⇒   [tex]x(1+0.263)=321[/tex]

⇒                    [tex]x=\frac{321}{1.263}[/tex]

⇒                       [tex]=254.15[/tex]

or,

⇒                    [tex]x\simeq 254[/tex]

Romans Food Market, located in Saratoga, New York, carries a variety of specialty foods from around the world. Two of the store's leading products use the Romans Food Market name: Romans Regular Coffee and Romans DeCaf Coffee. These coffees are blends of Brazilian Natural and Colombian Mild coffee beans, which are purchased from a distributor located in New York City. Because Romans purchases large quantities, the coffee beans may be purchased on an as-needed basis for a price 10% higher than the market price the distributor pays for the beans. The current market price is $0.47 per pound for Brazilian Natural and $0.62 per pound for Colombian Mild. The compositions of each coffee blend are as follows: Blend
Bean Regular DeCaf
Brazilian Natural 75% 40%
Columbian Mild 25% 60%
Romans sells the regular blend for $3.60 per pound and the Decaf blend for $4.40 per pound. Romans would like to place an order for the Brazilian and Columbian coffee beans that will enable the production of 1000 pounds of Romans Regular coffee and 500 pound of Romans DeCaf coffee. The production cost is $0.80 per pound for the Regular blend. Because of the extra steps required to produce DeCaf, the production cost for the DeCaf blend is $1.05 per pound. Packaging costs for both products are $0.25 per pound. Formulate a linear programming model that can be used to determine the pounds of Brazilian Natural and Columbian Mild that will maximize the total contribution to profit. What is the optimal solution and what is the contribution profit?

Answers

Answer:

z(max) =  2996.13  $

x₁  = 968      x₂  =  430       ( quantities of regular and Decaf coffee respectevely)

Total quantity of BN = 898 pounds

Total quantity of CM =  500 pounds

Step-by-step explanation:

Cost of the beans

Brazilian natural   =  Price market + 10 %    =  0.47 + 0.047  

BN  Cost   = 0.517 $/lb

Clombian Mild      =  Price market + 10 %    =  0.62  +  0.062

CM Cost   =  0.682  $/lb

Composition of the coffee blend

Regular coffee     0.75 BN  +  0.25 CM

De Caf   coffee     0.40 BN + 0.60  CM

PRICES

Regular Roman  =  3.60  $

Decaf                   =  4.40  $

Production costs:

Regular Roman  =  0.80  $/lb

Decaf                   =  1.05  $/lb

Packaging costs:   0.25  $/Lb       both

Profit  =  Price  -  cost

Profit  of regular coffee  =  3.60 - 0.80 - 0.25 -Cost of bean

for regular coffee cost of BN + CM

BN   is :   0.75*BN cost  = 0.75*0.517  =  0.38775       and

CM  is : 0.25*0.682   =   0.1705

Profit  of regular coffee  =  1.99175  $

Profit for Decaf coffee  =  4.4 - 1.05 - 0.25 - ( 0.517*0.4 + 0.6*0.682)

Profit for Decaf coffee  =  4.4  - 1.30 - 0.616

Profit for Decaf coffee  =  2.484  $

Let´s call  x₁  pounds of regular coffee   and  x₂   pounds of Decaf coffee then:

 Objective Function is:  

z  =  1.99175*x₁  +  2.484*x₂      to maximize

Subject to:

Availability of beans for 1000 pounds of Regular coffee means:

750 pounds of BN  +  250 pounds of CM

Availability of beans for 500 pounds of Decaf coffee means

200 pounds of BN  +   300 pounds of CM

Then   750 + 200  =  900 pounds of BN

And     250 + 300 =  550  pounds of CM

Availability of beans for 1000 pounds of Decaf  coffee correspond to

0.75 *x₁  +  0.40*x₂  ≤ 900

Availability of beans for 500 pounds of Regular  coffee correspond to

0.25*x₁  +  0.60*x₂  ≤ 500

Then the model is:

z  =  1.99175*x₁  +  2.484*x₂      to maximize

Subject to:

0.75 *x₁  +  0.40*x₂  ≤ 900

0.25*x₁  +  0.60*x₂  ≤ 500

General constraints  x₁ ≥ 0      x₂ ≥   0  both integers

After 6 iterations optimal solution  ( maximum z) is

z(max) =  2996.13  $

x₁  = 968      x₂  =  430  

x₁   and  x₂  are quantities of  Regular and Decaf coffee respectively, to find out quantities of Brazilian Natural and Colombian Mild

we proceed as follows

Regular coffee is :  0.75*968 =   726 pounds of BN

Decaf coffee is :      0.40*430 =   172  pounds of BN

Total quantity of BN = 898 pounds

Regular coffee is :  0.25*968   =  242 pounds of CM

Decaf coffee is :  0.6*430  =  258 pounds of CM

Total quantity of CM =  500 pounds


A driver must decide whether to buy a new car for $24,000 or lease the same car over a four-year period. Under the terms of the lease, she can make a down payment
of $3000 and have monthly payments of $150. At the end of the four years, the leased car has a residual value (the amount she pays if she chooses to buy the car at
the end of the lease period) of $11,000. Assume she can sell the new car at the end of the four years at the same residual value. Is it less expensive to buy or
to lease?

Answers

Answer:

3000 is the answer this question.

Other Questions
2.7.2 : Checkup - Practice Problems Which of the following best describes the state of the US economy at the end of World War 11?It was starting to come out of the Great Depression.It was out of the Great Depression and doing better.It was one of the most powerful economies in the world.It was entering another recession Can someone help me? Misusing words, or choosing the wrong words for the type of writing you create can confuse your reader. True False why is it wrong to leave our light on Give me a brief description of your overall summary of slavery in the 20th century . Also implement your feelings toward the subject. A rectangular floor of area 360 m2 is going to be tiled. Each tile is rectangular, and has an area of 240 cm2. An exact number of tiles can be put into the space. How many tiles will be needed?? Solve fast "Extravagance in feast, festival and other occasions obstructs the development of society." Do you agree with the statement? Write you opinion. 10. Football: A kicker on a professional footballteam made 45 of 48 field goal attempts.a. What percent of his attempts did hemake?b. What percent did he miss? Would you keepthis player on your team or trade for a newkicker? scientists are seen very busy in designing the solar power equipments, why? a vehicle starts from rest and has an acceleration of 2 metre per second square how long does it take to gain the speed of 20 metre per second QuestionThe quotient of a number and 5 has a result of 2. What is the number? ToolsSelect the correct answerRead the adapted excerpt from Gulliver's Travels by Jonathan SwiftBut I should have mentioned that before the principal person began his oration, he cried out three times, Langro debul san these words,and the former, were afterwards repeated, and explained to me), Whereupon immediately about ity of the inhabitants come and cut thestrings that fastened the left side of my head, which gave me the liberty of turning it to the right, and of observing the person and gesture ofhim that was to speak. He appeared to be of a middle age, and taller than any of the other three who attended him, whereof one was apage that held up his train, and seemed to be somewhat longer than my middle finger, the other two stood one on each side, to supporthim. He acted every part of an orator, and I could observe many periods of threatenings, and others of promises, pity, and kindnessWhich choice provides the best objective summary of the excerpt?OA The narrator is bound but an orator comes out and orders histles to be cut. This orator seems to be the leader as he isdressed formally and has others to support him. All follow his commandsOB. The narrator, tied down, watches as a leader of the group orders histles to be cut so that he can turn his head. This leaderseems to issue threats but also shows kindness,OC. The narrator is very confused as he is tied and cannot move, the inhabitants are speaking a language he does not understand,and now an oratoris both threatening him and showing pityOD. The narrator is tied down and cannot move until a leader orders his ties be cut. This leader, whom the narrator cannotunderstand, seems to show a variety of different moods,21 Edmentum. Al Each of the following reactions were in equilibrium when the pressure of their containers was doubled. Chose which way the reaction shifted after the pressure change: 2NH3 (g)No(g) +3H2(g) [Select ] 2Na3 PO4 (aq) + 3CaCl2 (aq) Ca3 (PO4 )2 (s) + 6NaCl(aq) [Select ] 2C0(g)+O2 (g)->2CO2 (g) [Select ] 2H1(g) H2(g) + 12(g) Neither What is the sum of the coefficients of the balanced equation for the following reaction: FeCl2(aq) K2Cr2O7(aq) HCl(aq) ---> CrCl3(aq) FeCl3(aq) KCl(aq) H2O(l) Suppose Skyler invests in an annuity that pays 3.2% annual interest, compounded monthly. If she contributes $155 every month for 10 years, how much interest would she earn during that time Which of the following expressions would represent a class of 42 students divided equally into 7 groups? Can someone help solve the problems 2-4 The cost of renting a car is $46/week plus $0.25/mile traveled during that week. An equation to represent the cost would be y = 46 + 0.25x, where x is the number of miles traveled.what is your cost if you travel 59 miles cost: 60.75if your cost Is $66.00, how many miles were you charged for traveling?miles: ?you have a max of $100 to spend on a car rental. what would be the maximum number of miles you could Travel? max miles: ? 11NaSodium22.99What does 22.99 stand for